Last visit was: 24 Apr 2024, 07:31 It is currently 24 Apr 2024, 07:31

Close
GMAT Club Daily Prep
Thank you for using the timer - this advanced tool can estimate your performance and suggest more practice questions. We have subscribed you to Daily Prep Questions via email.

Customized
for You

we will pick new questions that match your level based on your Timer History

Track
Your Progress

every week, we’ll send you an estimated GMAT score based on your performance

Practice
Pays

we will pick new questions that match your level based on your Timer History
Not interested in getting valuable practice questions and articles delivered to your email? No problem, unsubscribe here.
Close
Request Expert Reply
Confirm Cancel
SORT BY:
Date
Tags:
Difficulty: 505-555 Levelx   Strengthenx               
Show Tags
Hide Tags
User avatar
Manager
Manager
Joined: 16 Feb 2011
Posts: 142
Own Kudos [?]: 970 [139]
Given Kudos: 9
Send PM
Most Helpful Reply
User avatar
Retired Moderator
Joined: 20 Dec 2010
Posts: 1114
Own Kudos [?]: 4702 [40]
Given Kudos: 376
Send PM
Tutor
Joined: 16 Oct 2010
Posts: 14817
Own Kudos [?]: 64894 [23]
Given Kudos: 426
Location: Pune, India
Send PM
General Discussion
User avatar
Manager
Manager
Joined: 09 Sep 2011
Status:Enjoying the MBA journey :)
Posts: 120
Own Kudos [?]: 229 [7]
Given Kudos: 16
Location: United States (DC)
Concentration: General Management, Entrepreneurship
GMAT 1: 710 Q49 V38
WE:Corporate Finance (Other)
Send PM
Re: In many corporations, employees are being replaced by automated equipm [#permalink]
4
Kudos
3
Bookmarks
Hi

Option B talks about profits for corporations that have failed to automate. However, clearly the argument mentions only those corporations wherein automation is under way. Also, the argument never mentions profit as an aspect and is just concerned with people losing their jobs to automation.

Option A clearly strengthens the argument by eliminating the possibility that the workers who are losing jobs to automation might find new jobs without any problems. If it were the case, then the argument might be weakened.

Hope this helps.

Cheers!
User avatar
Manager
Manager
Joined: 09 Nov 2011
Posts: 94
Own Kudos [?]: 127 [3]
Given Kudos: 16
Send PM
Re: In many corporations, employees are being replaced by automated equipm [#permalink]
3
Kudos
Author's Argument: Companies eventually need to pay more tax.
Why: If unemployment increases, govt will increase taxes to pay off unemployement.

Hence anything that points to increase in unemployment increase will be a right answer.

Why not B - There is no mention of profit in the argument. Hence we cannot relate profit to taxes and also, the author is only concerned about the companies that are moving to automation and nothing about the companies that are not doing so. Option B is irrelevant.
User avatar
Manager
Manager
Joined: 14 Sep 2014
Posts: 98
Own Kudos [?]: 136 [9]
Given Kudos: 236
Concentration: Technology, Finance
WE:Analyst (Other)
Send PM
Re: In many corporations, employees are being replaced by automated equipm [#permalink]
7
Kudos
2
Bookmarks
The argument claims that the increase of layoffs will directly result in increased taxes for government assistance. The assumption is that those being laid off will not be able to find another job; if they could find another job, the number of unemployment claims would decrease. The argument is only focused on whether firms that are automating will pay more taxes for government assistance. The incorrect answers are trying to distract you.

A. Many workers who have already lost their jobs to automation have been unable to find new jobs. This is correct. If they could find another job, unemployment claims would decrease.
B. Many corporations that have failed to automate have seen their profits decline. profits are not relevant
C. Taxes and unemployment insurance are paid also by corporations that are not automating. firms that don't automate are not relevant
D. Most of the new jobs created by automation pay less than the jobs eliminated by automation did. difference in pay is not relevant
E. The initial investment in machinery for automation is often greater than the short-term savings in labor costs. return on investment is not relevant; we just want to know if these firms will pay more taxes
User avatar
Intern
Intern
Joined: 25 Apr 2013
Posts: 45
Own Kudos [?]: 33 [0]
Given Kudos: 12
Send PM
Re: In many corporations, employees are being replaced by automated equipm [#permalink]
I have a question. Is option A an assumption also? Because going by "negation technique" for assumptions, if option A is negated, in other words, if many workers who have lost their jobs to automation are able to find new jobs, then the conclusion of the argument no longer is valid.
Intern
Intern
Joined: 17 May 2016
Posts: 18
Own Kudos [?]: 62 [0]
Given Kudos: 38
GMAT 1: 740 Q46 V46
Send PM
Re: In many corporations, employees are being replaced by automated equipm [#permalink]
Hi All,

Sorry in advance if my question seems stupid :

While reading, I had the impression that there were no conclusion, only a bunch of statements.
Since the only affirmative sentence was the first, I thought that the conclusion was the following : Replacement of workers ==> Savings
I thought that the second part However it is possible that actually it costs money was only an hypothesis, a possibility.

Then my question is the following : How can you consider a probabability, without any affirmation (introduced by however), a conclusion ?

Thanks in advance for your kind help,
Manager
Manager
Joined: 30 Dec 2015
Posts: 58
Own Kudos [?]: 118 [3]
Given Kudos: 173
GPA: 3.92
WE:Engineering (Aerospace and Defense)
Send PM
Re: In many corporations, employees are being replaced by automated equipm [#permalink]
2
Kudos
1
Bookmarks
nickimonckom wrote:
Hi All,

Sorry in advance if my question seems stupid :

While reading, I had the impression that there were no conclusion, only a bunch of statements.
Since the only affirmative sentence was the first, I thought that the conclusion was the following : Replacement of workers ==> Savings
I thought that the second part However it is possible that actually it costs money was only an hypothesis, a possibility.

Then my question is the following : How can you consider a probabability, without any affirmation (introduced by however), a conclusion ?

Thanks in advance for your kind help,


Focus on whats highlighted:
However,many workers who lose their jobs to automation will need government assistance to survive, and the same corporations that are laying people off will eventually pay for that assistance through increased taxes and unemployment insurance payments.

Here the author is confirming that companies will eventually have to pay for unemployment. But what if the employees who lost their jobs to automation find jobs in another company? Then the company wont have to pay for unemployment.
But if the employees who lost their jobs to automation are unsuccessful in finding a job? Then these companies will have to pay for them through increased taxes and insurance.

Hope this makes sense.
Manager
Manager
Joined: 21 Mar 2019
Posts: 66
Own Kudos [?]: 88 [0]
Given Kudos: 1049
Location: India
GMAT 1: 640 Q45 V35
GMAT 2: 660 Q47 V34
Send PM
Re: In many corporations, employees are being replaced by automated equipm [#permalink]
Hello experts VeritasKarishma, GMATNinjaTwo, @sayantanc2k,@AndrewN

I could not find the conclusion confidently and though that "many workers who lose their jobs to automation will need government assistance to survive" is conclusion and last sentence seems like supporting it. Obviously i got it wrong. Can you please explain how to find the conclusion when statements in passage are very neutral or seem like facts.

Thanks in advance.
Tutor
Joined: 16 Oct 2010
Posts: 14817
Own Kudos [?]: 64894 [0]
Given Kudos: 426
Location: Pune, India
Send PM
Re: In many corporations, employees are being replaced by automated equipm [#permalink]
Expert Reply
kadamhari825 wrote:
Hello experts VeritasKarishma, GMATNinjaTwo, @sayantanc2k,@AndrewN

I could not find the conclusion confidently and though that "many workers who lose their jobs to automation will need government assistance to survive" is conclusion and last sentence seems like supporting it. Obviously i got it wrong. Can you please explain how to find the conclusion when statements in passage are very neutral or seem like facts.

Thanks in advance.


Conclusion is the main idea. If you were to summarise what is said in one line, how would you do it? What does the author want to tell you?
That automation will not lead to companies saving money eventually. That is the whole crux of the argument.

The conclusion statement may not even be mentioned clearly in the argument. The author could go on and on giving data and supporting explanations but if she is asked, "So what are you trying to say?" then the one sentence that the author will say next will be your conclusion.
Manager
Manager
Joined: 16 Oct 2021
Posts: 149
Own Kudos [?]: 14 [0]
Given Kudos: 22
Location: Canada
Send PM
Re: In many corporations, employees are being replaced by automated equipm [#permalink]
GMATNinja, could you please clarify why E is not correct answer choice?
Intern
Intern
Joined: 28 Jul 2022
Posts: 4
Own Kudos [?]: 1 [0]
Given Kudos: 25
Send PM
Re: In many corporations, employees are being replaced by automated equipm [#permalink]
GMATNinja ScottTargetTestPrep TestPrepUnlimited
this question is terribly written in my opinion. Interpretation of the authors main conclusion from my perspective, no matter how many times I read it, is that these companies will eventually pay these fired workers assistance. The question, in turn, is asking what strengthens the argument? i.e. show me more support that these companies are going to be paying the cost for these workers and the money will be going into their pockets. If either of you you gave some perspective it'd make my day! This questions has been annoying me.
GMAT Club Verbal Expert
Joined: 13 Aug 2009
Status: GMAT/GRE/LSAT tutors
Posts: 6917
Own Kudos [?]: 63653 [4]
Given Kudos: 1773
Location: United States (CO)
GMAT 1: 780 Q51 V46
GMAT 2: 800 Q51 V51
GRE 1: Q170 V170

GRE 2: Q170 V170
Send PM
Re: In many corporations, employees are being replaced by automated equipm [#permalink]
3
Kudos
1
Bookmarks
Expert Reply
jg3swish wrote:
GMATNinja ScottTargetTestPrep TestPrepUnlimited
this question is terribly written in my opinion. Interpretation of the authors main conclusion from my perspective, no matter how many times I read it, is that these companies will eventually pay these fired workers assistance. The question, in turn, is asking what strengthens the argument? i.e. show me more support that these companies are going to be paying the cost for these workers and the money will be going into their pockets. If either of you you gave some perspective it'd make my day! This questions has been annoying me.

You're right that the conclusion is found in the last clause of the passage: "the same corporations that are laying people off will eventually pay for that assistance through increased taxes and unemployment insurance payments."

We're then asked to strengthen this argument, but it's important to remember WHY the author arrived at the conclusion in the first place. He/she believes that workers that lose their jobs (to automation) will need government assistance to survive.

With that in mind, here's (A):

Quote:
(A) Many workers who have already lost their jobs to automation have been unable to find new jobs.

At first glance, it may not seem like this strengthens the conclusion that corporations will eventually have to pay for assistance. But keep in mind that the logic of the argument is that workers will need government assistance to survive. Do we know that's necessarily true? Often, people that lose their jobs simply find another job, and they don't end up needing government assistance to survive. So, why is it the case here that government assistance will be needed?

(A) suggests that there's already a pattern at work. Workers that lose their jobs (because of automation) are not able to find new jobs. If that's true, then we have more reason to believe that these unemployed workers will need government assistance. And consequently, we have more reason to believe that the same corporations will end up paying "for that assistance through increased taxes and unemployment insurance payments."

So, (A) is the best answer choice.

I hope that helps!
Intern
Intern
Joined: 13 Apr 2020
Posts: 15
Own Kudos [?]: 11 [0]
Given Kudos: 185
Send PM
In many corporations, employees are being replaced by automated equipm [#permalink]
Hello GMATNinja

Even though option D in this question seems out of Scope, can't we argue that since the corporations know that they will have to pay for the taxes to the government, they have purposely kept the income of the new employees low so that the money they saved could be paid as taxes and that would strengthen the argument, as they are paying the money.

Posted from my mobile device
GMAT Club Verbal Expert
Joined: 13 Aug 2009
Status: GMAT/GRE/LSAT tutors
Posts: 6917
Own Kudos [?]: 63653 [3]
Given Kudos: 1773
Location: United States (CO)
GMAT 1: 780 Q51 V46
GMAT 2: 800 Q51 V51
GRE 1: Q170 V170

GRE 2: Q170 V170
Send PM
Re: In many corporations, employees are being replaced by automated equipm [#permalink]
3
Kudos
Expert Reply
ShashankSS3 wrote:
Hello GMATNinja

Even though option D in this question seems out of Scope, can't we argue that since the corporations know that they will have to pay for the taxes to the government, they have purposely kept the income of the new employees low so that the money they saved could be paid as taxes and that would strengthen the argument, as they are paying the money.

Posted from my mobile device

The simple fact that the new jobs created pay less than the old jobs does NOT strengthen the idea that the corporations will have to pay for the new assistance.

As you said, it's definitely possible that they want to save money to pay for the assistance. But it's equally (and probably more) likely that they're just paying their employees whatever market rate is and trying to make the highest profit possible. For that reason, we can't be sure that (D) strengthens the argument, and we can eliminate it.

I hope that helps!
Manager
Manager
Joined: 06 Sep 2017
Posts: 230
Own Kudos [?]: 138 [0]
Given Kudos: 154
Send PM
Re: In many corporations, employees are being replaced by automated equipm [#permalink]
DeeptiM wrote:
In many corporations, employees are being replaced by automated equipment in order to save money. However, many workers who lose their jobs to automation will need government assistance to survive, and the same corporations that are laying people off will eventually pay for that assistance through increased taxes and unemployment insurance payments.

Which of the following, if true, most strengthens the author's argument?

(A) Many workers who have already lost their jobs to automation have been unable to find new jobs.

(B) Many corporations that have failed to automate have seen their profits decline.

(C) Taxes and unemployment insurance are paid also by corporations that are not automating.

(D) Most of the new jobs created by automation pay less than the jobs eliminated by automation did.

(E) The initial investment in machinery for automation is often greater than the short-term savings in labor costs.




Similar question from OG 11 is posted here: https://gmatclub.com/forum/in-many-corp ... 77123.html



The answer is (a).

The author's argument is that corporations that lay off workers to automate will eventually pay for that assistance through increased taxes and unemployment insurance payments. Option (a) strengthens this argument by providing evidence that many workers who lose their jobs to automation are unable to find new jobs. This means that the corporations will be paying for government assistance for a long time, as these workers will be on unemployment for a long time.

The other options do not strengthen the author's argument as much. Option (b) is about corporations that have failed to automate. This is not relevant to the author's argument, which is about corporations that have automated. Option (c) is about taxes and unemployment insurance being paid by all corporations, not just those that are automating. This does not strengthen the author's argument, as it does not show that corporations that automate will pay more in taxes and unemployment insurance. Option (d) is about the wages of new jobs created by automation. This is not relevant to the author's argument, which is about the government assistance that will be paid to workers who lose their jobs to automation. Option (e) is about the initial investment in machinery for automation. This is not relevant to the author's argument, which is about the long-term costs of automation, such as government assistance.

Therefore, the answer is (a).
Manager
Manager
Joined: 11 Jan 2021
Posts: 70
Own Kudos [?]: 13 [0]
Given Kudos: 369
Location: India
Concentration: Strategy, Social Entrepreneurship
GMAT 1: 620 Q44 V35
GPA: 4
WE:Analyst (Internet and New Media)
Send PM
In many corporations, employees are being replaced by automated equipm [#permalink]
DeeptiM wrote:
In many corporations, employees are being replaced by automated equipment in order to save money. However, many workers who lose their jobs to automation will need government assistance to survive, and the same corporations that are laying people off will eventually pay for that assistance through increased taxes and unemployment insurance payments.

Which of the following, if true, most strengthens the author's argument?

(A) Many workers who have already lost their jobs to automation have been unable to find new jobs.

(D) Most of the new jobs created by automation pay less than the jobs eliminated by automation did.


MartyMurray KarishmaB GMATNinja

I was stuck between (A) and (D) and I chose (D).
(A) seems to be a pseudo-strengthener as it uses the word "Many". If it used "Most" or "more likely", I would've chosen this one and eliminated the other one
(D) seems to be a strengthener as I think it conveys that, even if automation created new jobs, most of them would've paid less than the jobs eliminated by automation, giving another reason that workers may be slightly more likely to need assistance than they would have previously. And, as long as it slights strengthens the argument, it could be considered to be the correct answer

Could you help me understand what's the best way to reason through these 2 options? As it is a sub-600 question, not getting it right is really troubling.

Should we ignore the effect qualifier words (such as, many, few, some, etc.) have if an option points out an important flaw in the argument as option (A) does?
Tutor
Joined: 16 Oct 2010
Posts: 14817
Own Kudos [?]: 64894 [2]
Given Kudos: 426
Location: Pune, India
Send PM
Re: In many corporations, employees are being replaced by automated equipm [#permalink]
2
Kudos
Expert Reply
SlowTortoise wrote:
DeeptiM wrote:
In many corporations, employees are being replaced by automated equipment in order to save money. However, many workers who lose their jobs to automation will need government assistance to survive, and the same corporations that are laying people off will eventually pay for that assistance through increased taxes and unemployment insurance payments.

Which of the following, if true, most strengthens the author's argument?

(A) Many workers who have already lost their jobs to automation have been unable to find new jobs.

(D) Most of the new jobs created by automation pay less than the jobs eliminated by automation did.


MartyMurray KarishmaB GMATNinja

I was stuck between (A) and (D) and I chose (D).
(A) seems to be a pseudo-strengthener as it uses the word "Many". If it used "Most" or "more likely", I would've chosen this one and eliminated the other one
(D) seems to be a strengthener as I think it conveys that, even if automation created new jobs, most of them would've paid less than the jobs eliminated by automation, giving another reason that workers may be slightly more likely to need assistance than they would have previously. And, as long as it slights strengthens the argument, it could be considered to be the correct answer

Could you help me understand what's the best way to reason through these 2 options? As it is a sub-600 question, not getting it right is really troubling.

Should we ignore the effect qualifier words (such as, many, few, some, etc.) have if an option points out an important flaw in the argument as option (A) does?


Our argument relies on the idea that workers who are removed because of automation will need assistance so the company will have to pay more taxes and unemployment insurance payments. Hence the company may not really save money overall. But this logic holds if these people who lose jobs do not get other jobs to support themselves.
Option (A) says that many of these people have not been able to find new work which makes it likely that the Govt will need to support them. And from where will the Govt get the money? From taxes so the company will need to pay more. Hence it strengthen the argument. The use of 'many' is enough to suggest that the taxes may increase substantially.
Option (D) is out of scope of this argument. What new employees are paid versus what old were paid doesn't matter. The argument is implying that only unemployed people will need assistance. We do not have to assume that new workers will be paid so low that they will need assistance too. Had they talked about the number of jobs lost vs. number of jobs created by automation, then that would have been relevant to the argument. Say, if automation were to create more jobs than it were to take away, then we could have weakened the argument because number of unemployed might have reduced actually.

Hence answer is (A)

Check this video for more on strengthen/weaken questions: https://youtu.be/Q5SG0ZZcHKA
Tutor
Joined: 11 Aug 2023
Posts: 821
Own Kudos [?]: 1407 [2]
Given Kudos: 74
GMAT 1: 800 Q51 V51
Send PM
Re: In many corporations, employees are being replaced by automated equipm [#permalink]
2
Kudos
Expert Reply
SlowTortoise wrote:
DeeptiM wrote:
In many corporations, employees are being replaced by automated equipment in order to save money. However, many workers who lose their jobs to automation will need government assistance to survive, and the same corporations that are laying people off will eventually pay for that assistance through increased taxes and unemployment insurance payments.

Which of the following, if true, most strengthens the author's argument?

(A) Many workers who have already lost their jobs to automation have been unable to find new jobs.

(D) Most of the new jobs created by automation pay less than the jobs eliminated by automation did.


MartyMurray KarishmaB GMATNinja

I was stuck between (A) and (D) and I chose (D).
(A) seems to be a pseudo-strengthener as it uses the word "Many". If it used "Most" or "more likely", I would've chosen this one and eliminated the other one

Doesn't the fact that many people are unable to find new jobs indicate that many people will need assistance? Do we really need it to be the case that most of the people are unable to find new jobs? We don't, right?

Quote:
Should we ignore the effect qualifier words (such as, many, few, some, etc.) have if an option points out an important flaw in the argument as option (A) does?

Generally, we should pay attention to qualifiers. For example, in this case, the fact that "many" people are unable to find new jobs provides strong support for the conclusion.

Quote:
(D) seems to be a strengthener as I think it conveys that, even if automation created new jobs, most of them would've paid less than the jobs eliminated by automation, giving another reason that workers may be slightly more likely to need assistance than they would have previously. And, as long as it slights strengthens the argument, it could be considered to be the correct answer

In GMAT Critical Reasoning, we have to keep in mind what specifically an argument is about.

In this case, the argument is about "many workers who lose their jobs to automation."

Yes, I agree that, in a way, (D) could be considered a strengthener. So, the key is to see that the question-writer used the language "lay off" and "unemployment benefits."

That language indicates that, though the question-writer could have communicated more clearly what the argument is about, it's really about people who end up without jobs. So, (D) doesn't apply because (D) is about what happens when people have new jobs.

Once you see that issue with (D), if you understand how (A) works, you quickly pick (A) and don't have any problem with this question even though it could be a little better constructed.
GMAT Club Bot
Re: In many corporations, employees are being replaced by automated equipm [#permalink]
Moderators:
GMAT Club Verbal Expert
6917 posts
GMAT Club Verbal Expert
238 posts
CR Forum Moderator
832 posts

Powered by phpBB © phpBB Group | Emoji artwork provided by EmojiOne